Difference between revisions of "2021 WSMO Accuracy Round Problems/Problem 3"

(Created page with "==Problem== If <math>f</math> is a monic polynomial of minimal degree with rational coefficients satisfying <math>f\left(3+\sqrt{5}\right)=0</math> and <math>f\left(4-\sqrt{7}...")
 
(Solution)
 
Line 4: Line 4:
 
==Solution==
 
==Solution==
 
If <math>3+\sqrt{5}</math> is a root of the polynomial, then <math>3-\sqrt{5}</math> is too. Similarly, if <math>4-\sqrt{7}</math> is a root of the polynomial, then so is <math>4+\sqrt{7}.</math> Thus, <math>f(x)=d(x)\left(x-\left(3+\sqrt{5}\right)\right)\left(x-\left(3-\sqrt{5}\right)\right)\left(x-\left(4-\sqrt{7}\right)\right)\left(x-\left(4+\sqrt{7}\right)\right),</math> for some polynomial <math>d.</math> To minimize the degree of <math>f,</math> we can set the degree of <math>d</math> to 0, which means that <math>d(x)=c.</math> This means that the leading coefficient of <math>f(x)=c.</math> Since <math>f</math> is monic, <math>c=1,</math> which means that <cmath>f=\left(x-\left(3+\sqrt{5}\right)\right)\left(x-\left(3-\sqrt{5}\right)\right)\left(x-\left(4-\sqrt{7}\right)\right)\left(x-\left(4+\sqrt{7}\right)\right)=</cmath><cmath>\left(\left(x-3\right)-\sqrt{5}\right)\left(\left(x-3\right)+\sqrt{5}\right)\left(\left(x-4\right)+\sqrt{7}\right)\left(\left(x-4\right)-\sqrt{7}\right)=</cmath><cmath>\left(\left(x-3\right)^{2}-\left(\sqrt{5}\right)^{2}\right)\left(\left(x-4\right)^{2}-\left(\sqrt{7}\right)^{2}\right)=</cmath><cmath>\left(x^{2}-6x+4\right)\left(x^{2}-8x+9\right)=x^{4}-14x^{3}+61x^{2}-86x+36.</cmath> We conclude that <cmath>|f(1)|=|1-14+61-86+36|=|-2|=\boxed{2}.</cmath>
 
If <math>3+\sqrt{5}</math> is a root of the polynomial, then <math>3-\sqrt{5}</math> is too. Similarly, if <math>4-\sqrt{7}</math> is a root of the polynomial, then so is <math>4+\sqrt{7}.</math> Thus, <math>f(x)=d(x)\left(x-\left(3+\sqrt{5}\right)\right)\left(x-\left(3-\sqrt{5}\right)\right)\left(x-\left(4-\sqrt{7}\right)\right)\left(x-\left(4+\sqrt{7}\right)\right),</math> for some polynomial <math>d.</math> To minimize the degree of <math>f,</math> we can set the degree of <math>d</math> to 0, which means that <math>d(x)=c.</math> This means that the leading coefficient of <math>f(x)=c.</math> Since <math>f</math> is monic, <math>c=1,</math> which means that <cmath>f=\left(x-\left(3+\sqrt{5}\right)\right)\left(x-\left(3-\sqrt{5}\right)\right)\left(x-\left(4-\sqrt{7}\right)\right)\left(x-\left(4+\sqrt{7}\right)\right)=</cmath><cmath>\left(\left(x-3\right)-\sqrt{5}\right)\left(\left(x-3\right)+\sqrt{5}\right)\left(\left(x-4\right)+\sqrt{7}\right)\left(\left(x-4\right)-\sqrt{7}\right)=</cmath><cmath>\left(\left(x-3\right)^{2}-\left(\sqrt{5}\right)^{2}\right)\left(\left(x-4\right)^{2}-\left(\sqrt{7}\right)^{2}\right)=</cmath><cmath>\left(x^{2}-6x+4\right)\left(x^{2}-8x+9\right)=x^{4}-14x^{3}+61x^{2}-86x+36.</cmath> We conclude that <cmath>|f(1)|=|1-14+61-86+36|=|-2|=\boxed{2}.</cmath>
 +
 +
~pinkpig

Latest revision as of 12:29, 23 December 2021

Problem

If $f$ is a monic polynomial of minimal degree with rational coefficients satisfying $f\left(3+\sqrt{5}\right)=0$ and $f\left(4-\sqrt{7}\right)=0,$ find the value of $|f(1)|$.

Solution

If $3+\sqrt{5}$ is a root of the polynomial, then $3-\sqrt{5}$ is too. Similarly, if $4-\sqrt{7}$ is a root of the polynomial, then so is $4+\sqrt{7}.$ Thus, $f(x)=d(x)\left(x-\left(3+\sqrt{5}\right)\right)\left(x-\left(3-\sqrt{5}\right)\right)\left(x-\left(4-\sqrt{7}\right)\right)\left(x-\left(4+\sqrt{7}\right)\right),$ for some polynomial $d.$ To minimize the degree of $f,$ we can set the degree of $d$ to 0, which means that $d(x)=c.$ This means that the leading coefficient of $f(x)=c.$ Since $f$ is monic, $c=1,$ which means that \[f=\left(x-\left(3+\sqrt{5}\right)\right)\left(x-\left(3-\sqrt{5}\right)\right)\left(x-\left(4-\sqrt{7}\right)\right)\left(x-\left(4+\sqrt{7}\right)\right)=\]\[\left(\left(x-3\right)-\sqrt{5}\right)\left(\left(x-3\right)+\sqrt{5}\right)\left(\left(x-4\right)+\sqrt{7}\right)\left(\left(x-4\right)-\sqrt{7}\right)=\]\[\left(\left(x-3\right)^{2}-\left(\sqrt{5}\right)^{2}\right)\left(\left(x-4\right)^{2}-\left(\sqrt{7}\right)^{2}\right)=\]\[\left(x^{2}-6x+4\right)\left(x^{2}-8x+9\right)=x^{4}-14x^{3}+61x^{2}-86x+36.\] We conclude that \[|f(1)|=|1-14+61-86+36|=|-2|=\boxed{2}.\]

~pinkpig